1
$\begingroup$

I have been given this problem and I have to decide whether it it true or false.

$$ (\log_2 n)^{\sqrt{ \log_2 n}} = \mathcal{O}(n^{50}) $$

This is my solution. Let

$$ \text{Let } f(n)= (\log_2 n)^{\sqrt{ \log_2 n}} \text{ and } g(n)=n^{50} \\ $$ Now define two new functions.

$$ \text{Let } f^{*}(n):= \log {f(n)} \text{ and } g^{*}(n):= \log {g(n)} \\ $$

So,

$$ f^{*}(n):= \log {f(n)} = (\log_2({\log_2 n}))\sqrt{\log_2 {n}} = \mathcal{O}((\log n)^{3/2}) \\ g^{*}(n):= \log {g(n)} = 50\log_2n = \mathcal{O}(\log n) \\ $$ Now if we examine the asymptotic behaviour of these two functions we find that $$ f^{*} = \Omega(g^{*}) $$ Thus we can conclude that $f = \Omega(g)$. Hence, the statement is false.

Is my solution correct? I feel intuitively that $n^{50}$ is much bigger than $(\log_2 n)^{\sqrt{ \log_2 n}}$

$\endgroup$
3
  • $\begingroup$ Your formula for $f^*$ is wrong. $$f^*(n)=\log(\log n)\sqrt{\log n}.$$ $\endgroup$ Commented Jan 31, 2023 at 1:55
  • $\begingroup$ Ahh what a silly mistake. I've updated it now. How does it look now? $\endgroup$ Commented Jan 31, 2023 at 2:11
  • 1
    $\begingroup$ No, remember $f=O(\cdots)$ only gives an upper bound, so while $f^*(n)=O(\log^{3/2} n),$ $f^*$ is actually much less. You need a lower bound for $f^*$ to get $\Omega(g^*).$ It is actually not hard to show that d$f^*=o(g^*).$ $\endgroup$ Commented Jan 31, 2023 at 2:36

1 Answer 1

3
$\begingroup$

It is true that $(\log_2(\log_2 n))\sqrt {\log_2 n}=O((\log_2 n)^{3/2})$ but this is not useful.

For all sufficiently large $m$ we have $\log_2 m=3\cdot\log_2 (m^{1/3})<3\cdot\frac { m^{1/3}}{3}=m^{1/3}.$ So with $m=\log_2 n$ we have $\log_2(\log_2 n)<(\log_2 n)^{1/3}$ for all sufficiently large $n.$

So $f^*(n)=O(\,(\log_2 n)^{1/3}\sqrt {\log_2 n}\;)=O(\,(\log_2 n)^{5/6}).$

So $f^*(n)/g^*(n)\to 0.$

The Landau big-O gives a "ceiling" on the kind of growth. In this Q we want a much lower ceiling on $f^*(n)$ than $(\log_2 n)^{3/2}.$

In general, if $r,s>0$ and $B>1$ then $\lim_{x\to\infty}\dfrac {(\log_B x)^r}{x^s}=0.$

$\endgroup$
1
  • 1
    $\begingroup$ FYI. Let $x=y^{r/s}.$ Then $\dfrac {(\log_B x)^r}{x^s}=$ $\dfrac {(\log_B (y^{r/s}))^r}{y^r} =$ $\left(\frac {\log_B (y^{r/s})}{y}\right)^r=$ $(r/s)^r\left(\frac {\log_B y}{y}\right)^r$ $\to 0$ as $y\to\infty.$ $\endgroup$ Commented Jan 31, 2023 at 5:43

You must log in to answer this question.

Start asking to get answers

Find the answer to your question by asking.

Ask question

Explore related questions

See similar questions with these tags.